MVT/Rolle's Theorem: Defining Intervals

  • Thread starter Thread starter pyrosilver
  • Start date Start date
  • Tags Tags
    Theorem
Click For Summary
The discussion focuses on defining intervals for the Mean Value Theorem (MVT) and Rolle's Theorem using the function f(x) = (x-4)^(2/3). For MVT to apply, the interval must be continuous and differentiable, which is satisfied for [5,8], as it does not include the non-differentiable point at x=4. Rolle's Theorem requires that f(a) = f(b) for endpoints a and b, and since f is not differentiable at 4, valid intervals must exclude this point, making [1,7] invalid. Ultimately, no interval satisfies Rolle's Theorem due to the behavior of the function around x=4, confirming that MVT does not apply on intervals that include this point. Understanding these conditions is crucial for correctly applying MVT and Rolle's Theorem.
pyrosilver
Messages
38
Reaction score
0

Homework Statement



Define interval where a) MVT applies, b) Rolle's applies, c) MVT doesn't apply. explain. (In this, we're saying Rolle's is f(a) = f(b), not f(a) = (f(b) = 0.

Homework Equations





The Attempt at a Solution

f(x) = (x-4)^(2/3)

a) for MVT, must be cont on [a,b] and diff on (a,b). It is not diff at x=4, but for interval [5,8] it should be fine, so a = [5,8]?
b) I'm confused for this one. f(1) = 2.08. f(7) = 2.08. However, x is not diff at 4, so [1,7] isn't a valid interval, right? What is a valid interval, one that let's Rolle's apply?
c) MVT doesn't apply on [1,7] because x is not diff at 4? Is that okay?

Please help!
 
Physics news on Phys.org
pyrosilver said:

Homework Statement



Define interval where a) MVT applies, b) Rolle's applies, c) MVT doesn't apply. explain. (In this, we're saying Rolle's is f(a) = f(b), not f(a) = (f(b) = 0.

Homework Equations





The Attempt at a Solution

f(x) = (x-4)^(2/3)

a) for MVT, must be cont on [a,b] and diff on (a,b). It is not diff at x=4, but for interval [5,8] it should be fine, so a = [5,8]?
Yes, any interval that does not include 4 works.

b) I'm confused for this one. f(1) = 2.08. f(7) = 2.08. However, x is not diff at 4, so [1,7] isn't a valid interval, right? What is a valid interval, one that let's Rolle's apply?
You are correct that [1, 7] does not work. You want to find some a, b, such that a and b are both less than 4 or both larger than 4 such that f(a)= f(b).
It is easy to see that f(x) is increasing for x> 4 and decreasing for x< 4. That is, in order that f(a)= f(b), either a= b or one is less than 4 and the other larger than 4. Neither gives a valid interval. Notice that if there were a valid interval, Rolle's theorem would say that f'(c)= 0 at some point inside that interval. And f' is NEVER 0 for this function. There is no such interval.

c) MVT doesn't apply on [1,7] because x is not diff at 4? Is that okay?
Yes, or any interval including 4.

Please help!
 
Question: A clock's minute hand has length 4 and its hour hand has length 3. What is the distance between the tips at the moment when it is increasing most rapidly?(Putnam Exam Question) Answer: Making assumption that both the hands moves at constant angular velocities, the answer is ## \sqrt{7} .## But don't you think this assumption is somewhat doubtful and wrong?

Similar threads

  • · Replies 2 ·
Replies
2
Views
2K
  • · Replies 1 ·
Replies
1
Views
2K
  • · Replies 1 ·
Replies
1
Views
2K
  • · Replies 3 ·
Replies
3
Views
2K
  • · Replies 2 ·
Replies
2
Views
4K
  • · Replies 16 ·
Replies
16
Views
2K
  • · Replies 26 ·
Replies
26
Views
2K
Replies
4
Views
2K
Replies
4
Views
2K
  • · Replies 1 ·
Replies
1
Views
2K